Author Archives: Hung Nguyen

Đề thi Học kì 1 Toán 10 PTNK năm 2017 (CS2)

Đề và lời giải: Thầy Nguyễn Tấn Phát

Bài 1. Giải các phương trình sau:
a) $(x+2)\sqrt{x^2-5}=x^2-4$
b) $x^2+8x+|x+4|+14=0$
Bài 2. Tìm $a$, $b$, $c$ biết hàm số $y=ax^2+bx+c$ có đồ thị được cho như hình sau.

Bài 3. Tìm $m$ để phương trình $(m-1)^2x^2 – 4(m+1)x+3=0$ có hai nghiệm, trong đó có một nghiệm gấp 3 lần nghiệm còn lại.
Bài 4. Tìm số nguyên $m$ sao cho hệ $\left{ \begin{array}{l}
mx-y=1 \
x+4(m+1)y=4m
\end{array} \right. $ có nghiệm duy nhất và là nghiệm nguyên.
Bài 5. Tính giá trị của biểu thức $P=\dfrac{16\cos ^3 a – \sin ^3 a + 5\cos a}{9\cos a + \sin ^3 a}$ khi $\tan a =3$.

Bài 6. Cho ba vectơ $\overrightarrow{a}$, $\overrightarrow{b}$, $\overrightarrow{c}$ bất kì. Xét tính đúng, sai của các mệnh đề sau:
a) $\left[ \left( \overrightarrow{a} \cdot \overrightarrow{b} \right) \overrightarrow{c} – \left( \overrightarrow{a} \cdot \overrightarrow{c} \right) \overrightarrow{b} \right] $ vuông góc với $\overrightarrow{a}$
b) $\left( \overrightarrow{a}\cdot \overrightarrow{b} \right) \overrightarrow{c} = \left( \overrightarrow{b} \cdot \overrightarrow{c} \right) \overrightarrow{a}$
Bài 7. Cho $\overrightarrow{u}= (1;-2)$, $\overrightarrow{v} = (x;y)$. Tìm $x$, $y$ sao cho $\overrightarrow{u}$, $\overrightarrow{v}$ cùng phương và $\overrightarrow{u} \cdot \overrightarrow{v}=-\dfrac{13}{2}$. Tính $|\overrightarrow{v}|$.
Bài 8. Cho tam giác $ABC$ với $A(-3;6)$, $B(1;-2)$, $C(6;3)$. Tìm tọa độ tâm $I$ và bán kính đường tròn ngoại tiếp của tam giác $ABC$.
Bài 9. Cho các điểm $M(-1;2m+3)$, $N(-4; 5m)$ và $P(-3; 3m+2)$. Tìm điều kiện cần và đủ của $m$ để $M$, $N$, $P$ là ba đỉnh của một tam giác. Khi đó chứng minh $\angle NMP$ là góc nhọn.

Lời giải
Bài 1.
a) Nghiệm của phương trình $x=\dfrac{9}{4}$
b) Phương trình tương đương: $(x+4)^2 + |x+4| -2=0$. Đặt $t= |x+4|$, phương trình trở thành $t^2 +t-2=0$.

Từ đó giải được tập nghiệm của phương trình là $S=\left\{ -3;-5 \right\} $
Bài 2. $(P):y=x^2 -4x+2$

Bài 3. Để phương trình có hai nghiệm phân biệt $x_1$, $x_2$ thì $\left\{ \begin{array}{l}
m \ne 1 \\
\Delta = m^2 +14m+1 >0
\end{array} \right. $

Theo Viete, ta có: $\left\{ \begin{array}{l}
x_1+x_2 = \dfrac{4(m+1)}{(m-1)^2} \quad (2) \\
x_1x_2= \dfrac{3}{(m-1)^2} \quad (3)
\end{array} \right. $

Vì nghiệm này gấp ba nghiệm kia nên từ $(2)$, ta tìm được $x_1$, $x_2$ theo $m$, sau đó thay vào $(3)$ giải được $m=0$ (nhận)

Vậy $m=0$

Bài 4. Để hệ có nghiệm duy nhất thì $D \ne 0 \Leftrightarrow m \ne -\dfrac{1}{2}$.

Tính $D_x$, $D_y$, suy ra $x=\dfrac{4}{2m+1}$, $y=\dfrac{2m-1}{2m+1}$.

Để nghiệm nguyên thì $2m+1 \in U(4)$ và $2m-1 \, \vdots \, 2m+1$. Từ đó suy ra $m \in \left\{ -1; 3 \right\} $
Bài 5. Vì $\cos a \ne 0$ nên chia cả tử và mẫu của $P$ cho $\cos ^3 a$, ta được:
$$P= \dfrac{16-\tan ^3 a + 5 \left( \tan ^2 a +1 \right) }{9 \left( \tan ^2 a +1 \right) + \tan ^3 a} = \dfrac{1}{3}$$
Bài 6.
a) Xét tích vô hướng:
$\left[ \left( \overrightarrow{a} \cdot \overrightarrow{b} \right) \overrightarrow{c} – \left( \overrightarrow{a} \cdot \overrightarrow{c} \right) \overrightarrow{b} \right] \overrightarrow{a} \\= |\overrightarrow{a}|\cdot |\overrightarrow{b} | \cdot \cos (\overrightarrow{a} , \overrightarrow{b} ) \cdot |\overrightarrow{c}|\cdot |\overrightarrow{a} | \cdot \cos (\overrightarrow{c} , \overrightarrow{a} ) – |\overrightarrow{c}|\cdot |\overrightarrow{a} | \cdot \cos (\overrightarrow{c} , \overrightarrow{a} ) \cdot |\overrightarrow{a}|\cdot |\overrightarrow{b} | \cdot \cos (\overrightarrow{a} , \overrightarrow{b} ) = 0$
Suy ra $\left[ \left( \overrightarrow{a} \cdot \overrightarrow{b} \right) \overrightarrow{c} – \left( \overrightarrow{a} \cdot \overrightarrow{c} \right) \overrightarrow{b} \right] $ vuông góc với $\overrightarrow{a}$
b) $\left( \overrightarrow{a}\cdot \overrightarrow{b} \right) \overrightarrow{c}$ cùng phương với $\overrightarrow{c}$; $\left( \overrightarrow{b} \cdot \overrightarrow{c} \right) \overrightarrow{a}$ cùng phương với $\overrightarrow{a}$

Xét trường hợp $\overrightarrow{a}$ không cùng phương với $\overrightarrow{c}$ thì mệnh đề trên sai. Vậy mệnh đề trên sai.

Bài 7. $\overrightarrow{v}=\left( -\dfrac{13}{10} ; \dfrac{13}{5} \right) \Rightarrow |\overrightarrow{v} | = \dfrac{13\sqrt{5}}{10}$
Bài 8. $I(1;3)$, $R=5$
Bài 9. Để $MNP$ là tam giác thì $m \ne 1$

$\cos NMP = \dfrac{2+(m-1)^2}{\sqrt{1+(m-1)^2} \sqrt{4+(m-1)^2}} >0, \; \forall m$ nên $\angle NMP$ là góc nhọn.

Đề thi Học kì 1 Toán 10 PTNK năm 2016 (CS1)

Đề và lời giải: Thầy Nguyễn Tấn Phát

Bài 1. (1 điểm) Tìm m để phương trình $\dfrac{(x-1)(x-3m)}{\sqrt{x-2}+1}=0$ vô nghiệm
Bài 2. (1 điểm) Gọi $(P)$ là đồ thị của hàm số: $y= x^2 + bx + c \, \, (b,c \in \mathbb{R} )$. Biết các điểm $A(1;-4)$, $B(2;-3)$, thuộc $(P)$. \
Tìm tọa độ giao điểm của $(P)$ và $(P’)$, với $(P’)$ là đồ thị của hàm số $y= (2x-1)^2 -4$
Bài 3. (1 điểm) Cho hệ phương trình: $\left\{ \begin{array}{l}
x+\dfrac{1}{m} \sqrt{y} =4 \
\dfrac{1}{m} x + \sqrt{y} = \dfrac{2}{m} + 2
\end{array} \right.$, với m là tham số và $m \ne 0$. Định m để hệ phương trình có nghiệm duy nhất.
Bài 4. (2 điểm) Giải các phương trình sau:
a) $\sqrt{2x+1}+\sqrt{x-3}=4$
b) $x+ \dfrac{3x}{\sqrt{x^2-9}}=\dfrac{35}{4}$
Bài 5. (1 điểm) Chứng minh đẳng thức: $\tan^2 a – \tan^2 b = \dfrac{\sin(a+b).\sin(a-b)}{\cos^2a.\cos^2b}$
Bài 6. (1 điểm) Cho tam giác $ABC$ có các đỉnh $A(-1;3)$, $B(-3;-3)$, $C(2;2)$. Chứng minh tam giác $ABC$ là tam giác vuông và tìm trực tâm tam giác $ABC$.
Bài 7. (3 điểm) Cho hình bình hành $ABCD$ với $AB=6a$, $AD=3a$, $\angle ABC =60^0$. Gọi $M,N$ thỏa: $\overrightarrow{MA}+2 \overrightarrow{MB}=\overrightarrow{0}$, $3 \overrightarrow{ND}+2 \overrightarrow{NC}=\overrightarrow{0}$.
a) Tính $\overrightarrow{AM}. \overrightarrow{AD}$.
b) Tính độ dài cạnh $AN$ theo $a$.
c) Gọi $G$ là trọng tâm tam giác $AMN$. Tìm $x$ và $y$ thỏa: $\overrightarrow{BG}= x \overrightarrow{BA} + y \overrightarrow{BD}$.

Lời giải
Bài 1. (1 điểm)
Điều kiện: $x \ge 2$
$\dfrac{(x-1)(x-3m)}{\sqrt{x-2}+1}=0$ (1)
$\Leftrightarrow (x-1)(x-3m)=0$
$\Leftrightarrow x-3m=0$ (2) vì $x \ge 2$
Để phương trình (1) vô nghiệm thì (2) phải vô nghiệm $\Leftrightarrow 3m<2 \Leftrightarrow m < \dfrac{2}{3}$

Bài 2. (1 điểm)
$A(1;-4)$, $B(2;-3)$ thuộc $(P)$, ta có:
$\left\{ \begin{array}{l}
1+b+c=-4\\
4+2b+c=-3
\end{array} \right.$
$\Leftrightarrow \left\{ \begin{array}{l}
b=-2\\
c=-3
\end{array} \right.$
Do đó $(P): y=x^2 – 2x -3$
Phương trình hoành độ giao điểm của $(P)$ và $(P’)$: $ x^2-2x-3=(2x-1)^2 -4$
$\Rightarrow\left[ \begin{array}{l}
x=0 \Rightarrow y=-3 \\
x=\dfrac{2}{3} \Rightarrow y=-\dfrac{35}{9}
\end{array} \right.$
Vậy giao điểm của $(P)$ và $(P’)$ là $(0;-3)$ và $\left( \dfrac{2}{3};-\dfrac{35}{9} \right)$

Bài 3. (1 điểm)
Ta có:
$D=1-\dfrac{1}{m^2}$
$Dx= \dfrac{4m^2-2m-2}{m^2}$
$Dy= \dfrac{2m-2}{m}$
Để hệ phương trình có nghiệm duy nhất thì:
$\left\{ \begin{array}{l}
D \ne 0 \\
y=\dfrac{Dy}{D} \ge 0
\end{array} \right.$
$\Rightarrow \left\{ \begin{array}{l}
m \ne \pm 1\\
\left[ \begin{array}{l}
m \ge 0 \\
m \le -1
\end{array} \right.
\end{array} \right. $
$\Rightarrow \left[ \begin{array}{l}
m < -1 \\ \left\{ \begin{array}{l} m > 0 \\
m \ne 1
\end{array} \right.
\end{array} \right.$

Bài 4. (2 điểm)
a) Điều kiện: $x \ge 3$.
Ta có $\sqrt{2x+1}+\sqrt{x-3} = 4 \Leftrightarrow 2\sqrt{2x^2-5x-3}=18 – 3x$
$\Leftrightarrow 4(2x^2-5x-3) = 9x^2-108x + 324$ ($x\leq 6$)
$\Leftrightarrow x^2-88x+332 = 0 \Leftrightarrow x = 4 (n), x = 84(l)$.
Vậy $S=\{4\}$.
b) Điều kiện: $x^2 \ge 9 \Rightarrow \left[ \begin{array}{l}
x \ge 3 \\
x \le -3
\end{array} \right.$
$x+ \dfrac{3x}{\sqrt{x^2-9}}=\dfrac{35}{4}$
$\Rightarrow x^2 + \dfrac{9x^2}{x^2-9} + \dfrac{6x^2}{\sqrt{x^2-9}}=\left( \dfrac{35}{4} \right)^2$
$\Rightarrow \dfrac{x^4}{x^2-9}+\dfrac{2.x^2.3}{\sqrt{x^2-9}} +9 = \left( \dfrac{35}{4} \right)^2 + 9$
$\Rightarrow \left( \dfrac{x^2}{\sqrt{x^2-9}} +3 \right)^2 = \dfrac{1369}{16}$
$\Rightarrow \left[ \begin{array}{l}
\dfrac{x^2}{\sqrt{x^2-9}} +3 = \dfrac{37}{4} \\
\dfrac{x^2}{\sqrt{x^2-9}} +3 = -\dfrac{37}{4} \quad \text{(loại)}
\end{array} \right.$
$\Rightarrow \dfrac{x^2}{\sqrt{x^2-9}} = \dfrac{25}{4}$
$\Rightarrow 16x^4=625.x^2-9.625$
$\Rightarrow \left[ \begin{array}{l}
x= \pm 5\\
x= \pm \dfrac{15}{4}
\end{array} \right.$
Thử lại nghiệm ta chọn $x=5$ hoặc $x=\dfrac{15}{4}$
Vậy $x=5$ hoặc $x=\dfrac{15}{4}$

Bài 5. (1 điểm)
$\tan^2 a – \tan^2 b \\
= \dfrac{\sin^2a}{\cos^2a}-\dfrac{\sin^2b}{\cos^2b} \\
= \dfrac{\sin^2a.\cos^2b – \sin^2b.\cos^2a}{\cos^2a.\cos^2b} \\
= \dfrac{(\sin a.\cos b+\sin b.\cos a)(\sin a.\cos b-\sin b.\cos a )}{\cos^2a.\cos^2b} \\
= \dfrac{\sin(a+b).\sin(a-b)}{\cos^2a.\cos^2b}$
Bài 6. (1 điểm)
$AB^2 =(-3+1)^2+(-3-3)^2=40$
$BC^2 =(2+3)^2+(2+3)^2=50$
$AC^2 =(2+1)^2+(2-3)^2=10$
$\Rightarrow BC^2=AB^2+AC^2$
Vậy tam giác $ABC$ vuông tại $A$ và có $A$ là trực tâm.

Bài 7. (3 điểm)
a) $MA = \dfrac{2}{3}AB = 4a$
$\angle BAD = 180^0-\angle ABC = 180^0-60^0 = 120^0 $
$\overrightarrow{AM}.\overrightarrow{AD}=|\overrightarrow{AM}|.|\overrightarrow{AD}|.\cos(\overrightarrow{AM},\overrightarrow{AD}) = 4a.3a.\cos 120^0 = -6a^2$
b) $DN = \dfrac{2}{3} CN=\dfrac{2}{5} CD
= \dfrac{2}{5}.6a=\dfrac{12}{5}a$
Áp dụng định lý cosin cho tam giác $ADN$, ta có:
\begin{align*}
AN^2&= AD^2+DN^2-2.AD.DN.\cos \angle ADN \\
&= (3a)^2 + \left( \dfrac{12}{5}a \right)^2 – 2.3a.\dfrac{12}{5}a.\cos 60^0 \\
&= \dfrac{189}{25} a^2
\end{align*}
Do đó $AN= \dfrac{3\sqrt{21}}{5}a$
c) \begin{align*}
\overrightarrow{BG} &= \overrightarrow{BE} + \overrightarrow{EG} \\
&= \dfrac{2}{3}\overrightarrow{BA} + \dfrac{1}{3} \overrightarrow{EN} \\
&= \dfrac{2}{3}\overrightarrow{BA} + \dfrac{1}{3} \left( \overrightarrow{EA}+\overrightarrow{AN} \right) \\
&= \dfrac{2}{3}\overrightarrow{BA} + \dfrac{1}{3}.\dfrac{1}{3}\overrightarrow{BA}+ \dfrac{1}{3} \left( \overrightarrow{AD} + \overrightarrow{DN} \right) \\
&= \dfrac{2}{3}\overrightarrow{BA} + \dfrac{1}{9} \overrightarrow{BA} + \dfrac{1}{3} \left( \overrightarrow{BD} – \overrightarrow{BA} \right) + \dfrac{1}{3}.\left( -\dfrac{2}{5} \right) \overrightarrow{BA} \\
&= \dfrac{14}{45}\overrightarrow{BA} + \dfrac{1}{3} \overrightarrow{BD}
\end{align*}
Vậy $x=\dfrac{14}{45}$ và $y=\dfrac{1}{3}$

Đề thi Học kì 1 Toán 10 PTNK năm 2019 (CS1)

Đề và lời giải: Thầy Nguyễn Tấn Phát

Bài 1. (1 điểm) Tìm $m$ để phương trình $\dfrac{m^2x+m}{x-1}=1$ có đúng một nghiệm.
Bài 2. (2 điểm) Giải các phương trình sau:
a) $4x-\left| 3x-2 \right| =x^2$
b) $\left( x^2 +x-2 \right) \left( \sqrt{5x-1}-7+2x \right) =0$
Bài 3. (1 điểm) Cho parabol $(P): y=ax^2+bx+c$. Tìm $a$, $b$, $c$ biết điểm $B(-1;4)$ thuộc $(P)$ và $S(0;3)$ là đỉnh của parabol.
Bài 4. (1,5 điểm) Cho hệ phương trình $\left\{ \begin{array}{l}
2mx-(m+1)y=m+1 \
(m-2)x-\dfrac{m}{2}y=-\dfrac{m}{2}-2
\end{array} \right. $
a) Tìm $m$ để hệ phương trình có nghiệm.
b) Tìm nghiệm $\left( x_0; y_0 \right) $ của hệ thỏa $x_0-y_0=-2$
Bài 5. (0,5 điểm) Rút gọn: $P=\dfrac{\sin \left( x+ \dfrac{\pi}{2} \right) + 2\cos (x+ \pi)}{\cos (\pi -x )}$.
Bài 6. (2 điểm) Hình bình hành $ABCD$ có $AB=a$, $AD=a\sqrt{3}$ và $\angle BAD = 30^\circ $
a) Tính $\overrightarrow{AB} \cdot \overrightarrow{AD}$ và độ dài đoạn $AC$.
b) Gọi $DE$ là đường cao của tam giác $ABD$ ($E$ thuộc đường thẳng $AB$). Tính $\overrightarrow{AB}\cdot \overrightarrow{AE}$ và độ dài đoạn $DE$.
Bài 7. (2 điểm) Trong mặt phẳng $Oxy$, cho $A(6;-2)$, $B(3;-1)$, $C(9;7)$.
a) Chứng minh $ABC$ là tam giác vuông và tìm $I$ thuộc trục tung sao cho $\overrightarrow{IB} \cdot \overrightarrow{AB} =10$.
b) Tính độ dài đoạn $AG$ với $G$ là trọng tâm tam giác $ABC$. Tìm điểm $K$ thuộc đường thẳng $d: y=x$ sao cho $\left| \overrightarrow{KB} + \overrightarrow{KC} \right| = 2\sqrt{5}$

Lời giải
Bài 1. (1 điểm)
a) Điều kiện: $x\ne 1$

Phương trình $(1) \Rightarrow \left( m^2-1 \right) x=-m-1 \quad (2)$

Phương trình $(1)$ có đúng một nghiệm khi và chỉ khi phương trình $(2)$ có đúng một nghiệm khác 1

$\Leftrightarrow \left\{ \begin{array}{l}
m^2-1 \ne 0 \\
x= \dfrac{-m-1}{m^2-1} \ne 1
\end{array} \right. \Leftrightarrow \left\{ \begin{array}{l}
m\ne 1 \\
m\ne -1 \\
m \ne 0
\end{array} \right. $

Bài 2. (2 điểm)
a)

  • Nếu $x\ge \dfrac{2}{3}$ ta có phương trình:
    $$4x-3x+2=x^2 \Leftrightarrow x^2-x-2=0 \Leftrightarrow \left[ \begin{array}{l}
    x=-1 \quad \text{(loại)} \\
    x=2 \quad \text{(nhận)}
    \end{array} \right. $$
  • Nếu $x < \dfrac{2}{3}$ ta có phương trình:
    $$4x+3x-2=x^2 \Leftrightarrow x^2-7x+2=0 \Leftrightarrow \left[ \begin{array}{l}
    x=\dfrac{7+\sqrt{41}}{2} \quad \text{(loại)} \\
    x=\dfrac{7-\sqrt{41}}{2} \quad \text{(nhận)}
    \end{array} \right. $$

Vậy phương trình có hai nghiệm $x=2$, $x=\dfrac{7-\sqrt{41}}{2}$
b) Điều kiện: $x\ge \dfrac{1}{5}$

$\left( x^2 +x-2 \right) \left( \sqrt{5x-1}-7+2x \right) =0 \Leftrightarrow \left[ \begin{array}{l}
x^2+x-2=0 \quad (2) \\
\sqrt{5x-1}=7-2x \quad (3)
\end{array} \right. $

$(1)\Leftrightarrow \left[ \begin{array}{l}
x=1 \quad \text{(nhận)} \\
x=-2 \quad \text{(loại)}
\end{array} \right. $

$(2)\Leftrightarrow \left\{ \begin{array}{l}
7-2x \ge 0 \\
5x-1 = 49 -28x+4x^2
\end{array} \right. \Leftrightarrow \left\{ \begin{array}{l}
x \le \dfrac{7}{2} \\
4x^2 – 33x + 50=0
\end{array} \right. \\
\Leftrightarrow \left\{ \begin{array}{l}
x\le \dfrac{7}{2} \\
\left[ \begin{array}{l}
x=2 \\
x=\dfrac{25}{4}
\end{array} \right.
\end{array} \right. \Leftrightarrow x=2 \quad \text{(nhận)}$
Vậy phương trình có 2 nghiệm $x=1$, $x=2$.

Bài 3. (1 điểm) $B(-1;4) \in (P) \Rightarrow 4=a-b+c$

$S(0;3)$ là đỉnh nên $3=c$ và $-\dfrac{b}{2a}=0 \Rightarrow b=0 \Rightarrow a=1$

Vậy $a=1$, $b=0$, $c=3$.

Bài 4. (1,5 điểm)
a) $D=\left| \begin{array}{*{20}{c}}
{2m}&{-(m+1)}\\
{m-2}&{-\dfrac{m}{2}}
\end{array} \right| = -m-2$

$D_x=\left| \begin{array}{*{20}{c}}
{m+1}&{-(m+1)}\\
{-\dfrac{m}{2}-2}&{-\dfrac{m}{2}}
\end{array} \right| = (m+1)(-m-2)$

$D_y=\left| \begin{array}{*{20}{c}}
{2m}&{m+1}\\
{m-2}&{-\dfrac{m}{2}-2}
\end{array} \right| = (-m-2)(2m-1)$
Hệ có nghiệm duy nhất khi và chỉ khi $D \ne 0 \Leftrightarrow -m-2 \ne 0 \Leftrightarrow m \ne -2$

Hệ có vô số nghiệm khi và chỉ khi $D=D_x=D_y=0\Leftrightarrow m=-2$

Khi đó hệ có vô số nghiệm là $(x,y)$ thỏa $-4x+y=-1$

Vậy hệ có nghiệm với mọi giá trị của $m$.
b)

  • TH1. Hệ có một nghiệm duy nhất khi và chỉ khi $D \ne 0\ne m \ne -2$Khi đó $x_0=m+1$, $y_0=2m-1$

    $x_0-y_0=-2 \Leftrightarrow m+1-2m+1=-2 \Leftrightarrow m=4$ (nhận)

    Khi đó nghiệm của hệ là $(5;7)$.

  • TH2. Hệ có vô số nghiệm khi và chỉ khi $D=D_x=D_y=0 \Leftrightarrow m=-2$Khi đó hệ có vô số nghiệm $\left( x_0;y_0 \right) $ thỏa $-4x_0+y_0=-1$

    Khi đó ta có hệ phương trình $\left\{ \begin{array}{l}
    x_0-y_0=-2 \\
    -4x_0+y_0=-1
    \end{array} \right. \Leftrightarrow \left\{ \begin{array}{l}
    x_0=1 \\
    y_0=3
    \end{array} \right. $

    Hệ có nghiệm là $(1;3)$.

Bài 5. (0,5 điểm) $\sin \left( x+ \dfrac{\pi}{2} \right) =\cos x$, $\cos (x+ \pi) = -\cos x$, $\cos (\pi -x) = – \cos x$

$P=\dfrac{\cos x – 2\cos x}{-\cos x} = 1$
Bài 6. (2 điểm)
a) $\overrightarrow{AB} \cdot \overrightarrow{AD} = AB\cdot AD \cdot \cos BAD = \dfrac{3}{2} a^2$

$\overrightarrow{AC} = \overrightarrow{AB} + \overrightarrow{AD}$

$AC^2 = \left( \overrightarrow{AB} + \overrightarrow{AD} \right) ^2 = AB^2 + AD^2 + 2\overrightarrow{AB} \cdot \overrightarrow{AD} = a^2 + 3a^2 + 3a^2 = 7a^2$

$\Rightarrow AC=\sqrt{7}a$
b) Đặt $\overrightarrow{AE} = x\cdot \overrightarrow{AB}$

$\overrightarrow{DE} = \overrightarrow{AE} – \overrightarrow{AD} = x\overrightarrow{AB}- \overrightarrow{AD}$

$DE \bot AB \Leftrightarrow \overrightarrow{DE} \bot \overrightarrow{AB} \Leftrightarrow \overrightarrow{DE} \cdot \overrightarrow{AB} =0 \\
\Leftrightarrow \left( x\overrightarrow{AB} – \overrightarrow{AD} \right) \cdot \overrightarrow{AB} = 0 \\
\Leftrightarrow xAB^2 – \overrightarrow{AB} \cdot \overrightarrow{AD} =0 \Leftrightarrow xa^2 – \dfrac{3}{2}a^2 =0 \Leftrightarrow x=\dfrac{3}{2}$

Vậy $\overrightarrow{AE} = \dfrac{3}{2} \overrightarrow{AB}$

$\overrightarrow{AB} \cdot \overrightarrow{AE} = \overrightarrow{AB} \cdot \left( \dfrac{3}{2} \overrightarrow{AB} \right) = \dfrac{3}{2} AB^2 = \dfrac{3}{2} a^2$

$DE^2 = \left( \dfrac{3}{2} \overrightarrow{AB} – \overrightarrow{AD} \right) ^2 = \dfrac{9}{4}AB^2 + AD^2 – 3\overrightarrow{AB} \cdot \overrightarrow{AD} = \dfrac{9}{4}a^2 + 3a^2 – \dfrac{9}{2}a^2 = \dfrac{3}{4}a^2 \\
\Rightarrow DE = \dfrac{\sqrt{3}}{2}a$

Hoặc cách khác.

Tam giác $ABE$ vuông tại $E$ có $\angle DAE = 30^\circ $ nên $\sin 30^\circ = \dfrac{DE}{AD} \Rightarrow DE=AD \cdot \sin 30^\circ = a\sqrt{3}\cdot \dfrac{1}{2} = \dfrac{a\sqrt{3}}{2}$

Bài 7. (2 điểm)
a) $\overrightarrow{AB}(-3;1)$, $\overrightarrow{AC}(3;9)$

$\overrightarrow{AB} \cdot \overrightarrow{AC} = -3\cdot 3 + 1\cdot 9 =0 \Rightarrow \overrightarrow{AB} \bot \overrightarrow{AC} \Rightarrow \triangle ABC$ vuông tại $A$.

Đặt $I(0;i)$

$\overrightarrow{IB}(3;-1-i)$, $\overrightarrow{AB}(-3;1)$

$\overrightarrow{IB} \cdot \overrightarrow{AB} =10 \Leftrightarrow 3\cdot (-3) + (-1-i)\cdot 1= 10 \Leftrightarrow i=-20$

Vậy $I(0;-20)$.
b) $G\left( 6; \dfrac{4}{3} \right) $

$\overrightarrow{AG}\left( 0; \dfrac{10}{3} \right) \Rightarrow AG= \dfrac{10}{3}$

Đặt $K(k;k)$

$\overrightarrow{KB}(3-k;-1-k)$, $\overrightarrow{KC}(9-k;7-k)$

$\overrightarrow{KB}+ \overrightarrow{KC} = (12-2k;6-2k)$

$\left| \overrightarrow{KB} + \overrightarrow{KC} \right| = 2\sqrt{5} \Leftrightarrow (12-2k)^2 + (6-2k)^2 =20 \\
\Leftrightarrow k^2-9k+20=0 \Leftrightarrow \left[ \begin{array}{l}
k=4 \\
k=5
\end{array} \right. $

Vậy $K(4;4)$ hoặc $K(5;5)$.

Đề thi Học kì 1 Toán 10 PTNK năm 2020 (CS2)

Bài 1. (2 điểm) Giải các phương trình:
a) $\dfrac{x^2 – 3x -4}{\sqrt{3-x}}=0$
b) $\sqrt{x+2}= \sqrt{2x+5} – \sqrt{3-x}$
Bài 2. (1 điểm) Tìm tọa độ đỉnh $I$ của parabol $(P): y= ax^2 + bx+ c \ (a \ne 0)$, biết parabol $(P)$ cắt trục hoành tại hai điểm có hoành độ lần lượt bằng 2 và 8, cắt trục tung tại điểm có tung độ bằng 8.
Bài 3. (1 điểm) Tìm $m$ để phương trình $\dfrac{x(2-x)}{\sqrt{2-x}} = \left( m^2 +1 \right) \sqrt{2-x}$ có nghiệm.
Bài 4. (1 điểm) Tìm $m$ để hệ phương trình $\left\{ \begin{array}{l}
(m+1)x-2y =m-1 \
m^2x-y = m^2 + 2m
\end{array} \right. $ có nghiệm duy nhất $\left( x_0; y_0 \right) $. Xác định một hệ thức liên hệ giữa $x_0$ và $y_0$ mà không phụ thuộc vào $m$.
Bài 5. (1 điểm) Cho góc $a$ thỏa $\tan \left( a + \dfrac{\pi}{2} \right) = -\sqrt{3}$. Tính giá trị của biểu thức:
$$P=\dfrac{\sin ^6 a + \cos ^6 a + 2\sin ^3 a \cdot \cos ^3 a}{\sin ^5 a \cdot \cos ^3 a + \sin ^3 a \cdot \cos ^5 a}$$
Bài 6. (2 điểm) Cho tam giác $ABC$ nhọn có độ dài cạnh $AB=5$. Gọi $H$ là chân đường cao hạ từ $A$ và $BH=3$, $CH=6$.
a) Tính $\overrightarrow{BA} \cdot \overrightarrow{BC}$ và độ dài $AC$.
b) Gọi $M$ là trung điểm của $AH$. Tính $\overrightarrow{MB} \cdot \overrightarrow{MC}$.
Bài 7. (2 điểm) Trong mặt phẳng $Oxy$, cho tam giác $ABC$ có $A(1;2)$, $B(-1;5)$, $C(3;2)$.
a) Tìm tọa độ trọng tâm $G$ và điểm $I$ thỏa $2\overrightarrow{IA} + 3\overrightarrow{IB} = 4\overrightarrow{IC}$.
b) Tìm tọa độ điểm $D$ biết $ABCD$ là hình thang có đáy $AB = \dfrac{3}{8}CD$.

Lời giải
Bài 1.
a) Điều kiện: $x<3$
$\dfrac{x^2 – 3x -4}{\sqrt{3-x}}=0 \Leftrightarrow (x+1)(x-4)=0 \Leftrightarrow \left[ \begin{array}{l}
x=-1 \quad (n) \\
x=4 \quad \ (l)
\end{array} \right. $
Vậy $S=\left\{ -1 \right\} $

b) $\sqrt{x+2}= \sqrt{2x+5} – \sqrt{3-x} \quad (1)$
Điều kiện: $-2 \le x \le 3$
$(1) \Leftrightarrow \sqrt{x+2} + \sqrt{3-x} = \sqrt{2x+5} \\
\Leftrightarrow 5 + 2\sqrt{-x^2+x+6} = 2x+5 \\
\Leftrightarrow \sqrt{-x^2+x+6} = x \quad (x\ge 0) \\
\Leftrightarrow -x^2 + x+6 = x^2 \\
\Leftrightarrow \left[ \begin{array}{l}
x=-\dfrac{3}{2} \quad (l) \\
x=2 \hspace{0.8cm} (n)
\end{array}\right.$

Vậy $S=\left\{ 2 \right\} $

Bài 2. (1 điểm) Theo đề ta có $(2;0)$, $(8;0)$, $(0;8)$ thuộc $(P)$ nên ta có hệ:

$\left\{ \begin{array}{l}
4a + 2b + c=0 \\
64a + 8b + c =0 \\
c=8
\end{array} \right. \Leftrightarrow \left\{ \begin{array}{l}
a=\dfrac{1}{2} \\
b=-5 \\
c=8
\end{array} \right. $

Suy ra $(P): y=\dfrac{1}{2}x^2 -5x+8$

Vậy tọa độ đỉnh $I$ của $(P)$ là $I\left( 5 ; -\dfrac{9}{2} \right) $
Bài 3. (1 điểm) ĐKXĐ: $x<2$
$\dfrac{x(2-x)}{\sqrt{2-x}} = \left( m^2 +1 \right) \sqrt{2-x} \Leftrightarrow x(2-x) = (m^2+1) (2-x)$
$\Leftrightarrow \left[ \begin{array}{l}
x=2 \ (l)\\
x=m^2 +1
\end{array}\right.$
Phương trình có nghiệm khi và chỉ khi $m^2+1 <2 \Leftrightarrow m^2 <1 \Leftrightarrow -1 < m <1$
Vậy khi $-1<m<1$ thì phương trình có nghiệm $x=m^2+1$.
Bài 4. (1 điểm)
Ta có:
$D=\left| \begin{array}{*{20}{c}}
{m+1}&{-2}\\
{m^2}&{-1}
\end{array}\right| = 2m^2 -m-1=(m-1)(2m+1)$

$D_x = \left| \begin{array}{*{20}{c}}
{m-1} & {-2}\\
{m^2 +2m} & {-1}
\end{array}\right| = 2m^2 +3m+1 = (2m+1)(m+1)$

$D_y=\left| \begin{array}{*{20}{c}}
{m+1} & {m-1}\\
{m^2} & {m^2 +2m}
\end{array}\right| = 4m^2 +2m = 2m(2m+1)$

Hệ phương trình có nghiệm duy nhất khi và chỉ khi $D\ne 0 \Leftrightarrow \left\{ \begin{array}{l}
m\ne 1\\
m\ne -\dfrac{1}{2}
\end{array}\right.$

Ta có: $\left\{ \begin{array}{l}
x_0 = \dfrac{D_x}{D} = \dfrac{m+1}{m-1}\\\\
y_0= \dfrac{D_y}{D} = \dfrac{2m}{m-1}
\end{array}\right. $

Ta có: $y_0 -x_0 = \dfrac{2m-m-1}{m-1} = 1$
Bài 5. (1 điểm) Ta có: $\tan \left(\alpha + \dfrac{\pi}{2}\right) = -\sqrt{3} \Leftrightarrow \tan \alpha = \dfrac{1}{\sqrt{3}}$

ĐKXĐ: $\cos \alpha \ne 0$

Chia cả tử và mẫu của $P$ cho $\cos \alpha ^8$ ta được:

$P=\dfrac{\sin ^6 a + \cos ^6 a + 2\sin ^3 a \cdot \cos ^3 a}{\sin ^5 a \cdot \cos ^3 a + \sin ^3 a \cdot \cos ^5 a}$

$=\dfrac{ \tan \alpha ^6 \cdot (\tan \alpha ^2 +1) + \tan \alpha ^2 +1 + 2\tan \alpha ^3 \cdot (\tan \alpha ^2 +1)}{\tan \alpha ^5 + \tan \alpha ^3}$

$=\dfrac{28+8\sqrt{3}}{3\sqrt{3}}$
Bài 6. (2 điểm)
a) Tam giác $ABH$ vuông tại $H$ nên $AH^2 = AB ^2 – BH^2 =16 \Rightarrow AH=4$

Tam giác $ACH$ vuông tại $H$ nên $AC^2 = AH^2 + CH^2 = 52 \Rightarrow AC=2\sqrt{13}$

Ta có: $CA^2 = \overrightarrow{CA}^2 = (\overrightarrow{BA} – \overrightarrow{BC} )^2 = BA^2 + BC^2 -2\overrightarrow{BA}\cdot \overrightarrow{BC}$

$\Rightarrow 52 = 25+ 81 – 2 \overrightarrow{BA} \cdot \overrightarrow{BC}$
$\Rightarrow \overrightarrow{BA} \cdot \overrightarrow{BC} = 27$
b) Tam giác $MBH$ vuông tại $H$ có: $MB^2 = MH^2 + BH^2 = 13 \Rightarrow MB =\sqrt{13}$

Tam giác $MCH$ vuông tại $H$ có: $MC^2 = MH^2 + CH^2 = 40 \Rightarrow MC = 2\sqrt{10}$

Ta có: $ CB^2 = (\overrightarrow{MB} – \overrightarrow{MC})^2 = MB^2 + MC^2 – 2\overrightarrow{MB} \cdot \overrightarrow{MC}$

$\Rightarrow 81 = 13 + 40 – 2\overrightarrow{MB} \cdot \overrightarrow{MC} \Rightarrow \overrightarrow{MB} \cdot \overrightarrow{MC} = -14$
Bài 7. (2 điểm)
a) Gọi $G(x_G; y_G)$

Ta có: $\left\{ \begin{array}{l}
x_G = \dfrac{x_A+x_B+x_C}{3} = 1\\
y_G = \dfrac{y_A + y_B + y_C}{3} =3
\end{array}\right. \Rightarrow G(1;3)$

Gọi $I(x_I; y_I)$

Khi đó $\overrightarrow{IA} = (1-x_I; 2-y_I)$, $\overrightarrow{IB} = (-1-x_I; 5-y_I)$, $\overrightarrow{IC} = (3-x_I; 2-y_I)$

Ta có: $2\overrightarrow{IA} + 3 \overrightarrow{IB} = 4\overrightarrow{IC} \Rightarrow \left\{ \begin{array}{l}
2(1-x_I) + 3(-1-x_I) = 4(3-x_I)\\
2(2-y_I) + 3(5-y_I) = 4(2-y_I)
\end{array}\right.$
$\Rightarrow \left\{ \begin{array}{l}
x_I = -13\\
y_I = 11
\end{array}\right.$

Vậy $I(-13; 11)$.
b) Gọi $D(x_D; y_D)$ khi đó $\overrightarrow{DC} = (3-x_D ; 2-y_D)$, $\overrightarrow{AB} = (-2; 3)$

Ta có: $AB // CD$ và $AB = \dfrac{3}{8}CD$

$ \Rightarrow \overrightarrow{AB} = \dfrac{3}{8} \overrightarrow{DC} \Rightarrow \left\{ \begin{array}{l}
-2 = \dfrac{3}{8} (3-x_D)\\
3 = \dfrac{3}{8} (2-y_D)
\end{array}\right. \Rightarrow \left\{ \begin{array}{l}
x_D = \dfrac{25}{3}\\
y_D = -6
\end{array}\right.$

Vậy $D\left(\dfrac{25}{3}; -6\right)$

Đề thi Học kì 1 Toán 10 PTNK năm 2020 (CS1)

Đề thi và đáp án HK1 môn toán 10 trường PTNK (CS1)

Năm học 2020 – 2021

Thực hiện: Thầy Nguyễn Tấn Phát – GV PTNK

Bài 1. (2 điểm) Giải các phương trình:
a) $\dfrac{{{x^4} – 10{x^2} + 9}}{{\sqrt {x – 2} }} = 0$
b) $x\sqrt {{x^2} – x + 3} = x\left( {x – 6} \right)$
Bài 2. (1 điểm) Tìm $m$ để phương trình $\dfrac{1}{x} + \dfrac{{m + x}}{{x – 1}} = 1$ có nghiệm duy nhất.
Bài 3. (1 điểm) Chứng minh
$$\left[ {\cos 2\pi – \cos \left( {2\pi + x} \right)} \right]\left[ {1 + {{\tan }^2}\left( {\frac{\pi }{2} – x} \right)} \right] = \frac{1}{{1 + \cos x}}$$
Bài 4. (1 điểm) Cho hệ phương trình $\left\{ \begin{array}{l}
mx – \left( {m + 1} \right)y = 1\
\left( {2 – m} \right)x + \left( {m – 3} \right)y = 3 – 2m
\end{array} \right.$ ($m$ là tham số).
a) Tìm $m$ để hệ có nghiệm duy nhất $\left( x_0; y_0 \right) $.
b) Chứng minh $x_0^2 – y_0^2 – 2{x_0} = – 1$
Bài 5. (1 điểm) Gọi $(P)$ là đồ thị của hàm số $y = {x^2} + 2x – m$. Biết $(P)$ cắt trục tung tại điểm có tung độ là 4. Tìm m và tọa độ đỉnh của $(P)$.
Bài 6. (2 điểm) Cho hình bình hành ABCD có $AD = a$, $AB = 2a$ và $\widehat {DAB} = 120^\circ $.
a) Tính $\overrightarrow{DA} \cdot \overrightarrow{AB}$. Chứng minh $AB^2 – AD^2 = \overrightarrow {AC} \cdot \overrightarrow {DB} $
b) Gọi $H$ là hình chiếu vuông góc của $A$ trên $DB$. Tính $\overrightarrow{DH} \cdot \overrightarrow{DA}$.
Bài 7. (2 điểm) Trong mặt phẳng $Oxy$, cho tam giác $ABC$ có $A(1;6)$, $B(6;5)$, $C(6;1)$.
a) Tìm tọa độ $M$ sao cho $\overrightarrow {CM} = \overrightarrow {CA} – \overrightarrow {CB} $
b) Đường tròn ngoại tiếp tam giác $ABC$ cắt trục tung tại hai điểm phân biệt $E$, $F$. Tìm tọa độ tâm đường tròn ngoại tiếp tam giác $ABC$. Tìm toạ độ $E$ và $F$.

Lời giải

Bài 1.
a) $\dfrac{{{x^4} – 10{x^2} + 9}}{{\sqrt {x – 2} }} = 0 \quad (1) $
Điều kiện: $x>2$
$(1) \Leftrightarrow {x^4} – 10{x^2} + 9 =0 \Leftrightarrow \left[ \begin{array}{l}
x=1 \quad (l) \\
x=-1 \quad (l) \\
x=3 \quad (n) \\
x=-3 \quad (l)
\end{array} \right. $
Vậy $S=\left\{ 3 \right\} $
b) $x\sqrt{x^2-x+3} = x(x-6)$ (NX: $x^2 -x+3 >0$, $\forall x\in \mathbb{R}$)
$\Leftrightarrow \left[ \begin{array}{l}
x=0\\
\sqrt{x^2 -x +3 } = x-6 \ (*)
\end{array}\right. $
$(*)\Leftrightarrow \left\{ \begin{array}{l} x-6\ge 0\\
x^2 -x +3 = (x-6)^2
\end{array}\right. $
$\Leftrightarrow \left\{ \begin{array}{l}
x\ge 6\\
x=3
\end{array}\right. $
$\Leftrightarrow x\in \emptyset$
Vậy $S=\left\{ 0\right\} $

Bài 2. (1 điểm) ĐKXĐ: $x\ne 0$, $x\ne 1$

Phương trình trở thành: $(m+2)x=1$

Phương trình có nghiệm duy nhất khi và chỉ khi $\left\{ \begin{array}{l}
m+2\ne 0\\\\
\dfrac{1}{m+2}\ne 0\\\\
\dfrac{1}{m+2}\ne 1
\end{array}\right. $
$\Leftrightarrow \left\{ \begin{array}{l}
m\ne -2\\
m\ne -1
\end{array}\right. $

Vậy $m\ne -2$ và $m\ne -1$ thì phương trình có nghiệm duy nhất $x=\dfrac{1}{m+2}$
Bài 3. (1 điểm)
$VT= \left[ {\cos 2\pi – \cos \left( {2\pi + x} \right)} \right]\left[ {1 + {{\tan }^2}\left( {\dfrac{\pi }{2} – x} \right)} \right] $

$= (1-\cos x) (1+\cot^2 x)$

$ = (1-\cos x) \cdot \dfrac{1}{\sin^2 x}$

$= (1-\cos x )\cdot \dfrac{1}{1-\cos^2 x}$

$=\dfrac{1}{1+\cos x}=VP$
Bài 4. (1 điểm)
a) Ta có:
$D=\left| \begin{array}{*{20}{c}}
{m}&{-(m+1)}\\
{2-m}&{m-3}
\end{array}\right| = 2(1-m)$

$D_x = \left| \begin{array}{*{20}{c}}
{1} & {-(m+1)}\\
{3-2m} & {m-3}
\end{array}\right| = 2m(1-m)$

$D_y=\left| \begin{array}{*{20}{c}}
{m} & {1}\\
{2-m} & {3-2m}
\end{array}\right| = -2(m-1)^2$

Hệ phương trình có nghiệm duy nhất khi và chỉ khi $D\ne 0 \Leftrightarrow m\ne 1$
b) Ta có: $\left\{ \begin{array}{l}
x_0 = \dfrac{D_x}{D} = m\\\\
y_0= \dfrac{D_y}{D} = m-1
\end{array}\right. $

Ta có: $x_0^2 – y_0^2 -2x_0 = m^2 – (m-1)^2 -2m =-1$
Bài 5. (1 điểm) Thay $M(0;4)$ vào $(P)$, ta có: $4=-m \Leftrightarrow m=-4$
Tọa độ đỉnh $I( -1;3)$
Bài 6. (2 điểm)
a) Ta có: $\overrightarrow{DA} \cdot \overrightarrow{AB} = -\overrightarrow{AD} \cdot \overrightarrow{AB} = – AD \cdot AB \cdot \cos 120^\circ = a^2$

Ta có: $AB^2 – AD^2 = \left( \overrightarrow{AB}\right) ^2 – \left( \overrightarrow{ AD}\right) ^2 $

$= \left( \overrightarrow{AB} – \overrightarrow{AD}\right) \left( \overrightarrow{AB} + \overrightarrow{AD} \right) = \overrightarrow{DB} \cdot \overrightarrow{AC}$
b) Đặt $\overrightarrow{DH} =x\overrightarrow{DB}$

Ta có: $\overrightarrow{AH} = x\overrightarrow{AB} + (1-x)\overrightarrow{AD}$

Ta có: $\overrightarrow{AH} \cdot \overrightarrow{BD} = 0$

$\Leftrightarrow \left( x\overrightarrow{AB} + (1-x)\overrightarrow{AD}\right) \cdot \left( \overrightarrow{AD} – \overrightarrow{AB}\right) =0$

$\Leftrightarrow x (-a^2) -4xa^2 + (1-x)a^2 -(1-x)(-a^2) =0$

$\Leftrightarrow x=\dfrac{2}{7}$

Ta có: $\overrightarrow{DH} = \dfrac{2}{7} \overrightarrow{DB}$

$\Rightarrow \overrightarrow{DA} \cdot \overrightarrow{DH} = \dfrac{2}{7} \overrightarrow{DA} \cdot \overrightarrow{DB}$

$=\dfrac{2}{7} \overrightarrow{DA} \left( \overrightarrow{DA} + \overrightarrow{AB}\right) $

$=\dfrac{2}{7} \left( DA^2 + \overrightarrow{DA} \cdot \overrightarrow{AB}\right) $

$=\dfrac{4}{7}a^2$
Bài 7. (2 điểm)
a) Gọi $M(x;y)$

Ta có: $\overrightarrow{CM} = \overrightarrow{CA} – \overrightarrow {CB}$
$\Leftrightarrow \overrightarrow{CM} = \overrightarrow{BA}$
$\Leftrightarrow \left\{ \begin{array}{l}
x-6 = -5\\
y-1=1
\end{array}\right. $
$\Leftrightarrow \left\{ \begin{array}{l}
x=1\\
y=2
\end{array}\right. $

Vậy $M(1;2)$
b) Gọi $I(x_I;y_I)$ là tâm đường tròn ngoại tiếp tam giác $ABC$.

Ta có: $\left\{ \begin{array}{l}
IA = IB\\
IA = IC
\end{array}\right. $
$\Rightarrow \left\{ \begin{array}{l}
5x_I -y_I =12\\
(5-y_I)^2 = (1-y_I)^2
\end{array}\right. $
$\Rightarrow \left\{ \begin{array}{l}
x_I=3\\
y_I=3
\end{array}\right. $

Gọi $E(0;y_E)\in Oy$.

Ta có: $IA = IE \Rightarrow (3-y_E)^2 =4 \Rightarrow \left[ \begin{array}{l}
y_E =1\\
y_E =5
\end{array}\right. $

Vậy $E(0;1)$, $F(0;5)$ hoặc ngược lại.

Đáp án đề thi chọn đội dự tuyển lớp 10 năm 2016 – 2017

Bài 1: Cho $x,y,z$ là các số thực dương thoả mãn $x+y+z=1$. Chứng minh rằng:

$$\dfrac{x^4}{x^3+y^2+z^2}+\dfrac{y^4}{y^3+z^2+x^2}+\dfrac{z^4}{z^3+x^2+y^2}\ge \dfrac{1}{7}.$$

Bài 2: Tìm tất cả các hàm số $f:\mathbb N^* \rightarrow \mathbb N^*$ thoả mãn đồng thời các điều kiện:

i/ $f(mn)=f(m)f(n)\ \forall m,n \in \mathbb N^*$.

ii/ $f(m)+f(n)$ chia hết cho $m+n$ $\forall m,n \in \mathbb N^*$.

iii/ $f(2017)=2017^3$.

Bài 3. Cho đường tròn $(O)$ và dây cung $AB$ cố định. $C$ là một điểm thay đổi trên cung lớn $AB$ sao cho tam giác $ABC$ nhọn. Gọi $I,I_a,I_b$ lần lượt là tâm đường tròn nội tiếp, tâm đường tròn bàng tiếp $\angle BAC$ và $\angle ABC$ của tam giác $ABC$.

a/ Gọi $M$ đối xứng với $I$ qua $O$. Chứng minh rằng tam giác $MI_{a}I_{b}$ cân.

b/ Gọi $H,K$ lần lượt là hình chiếu của $I_a,I_b$ trên $OI$. Đường thẳng qua $H$ vuông góc với $BI_a$ và đường thẳng qua $K$ vuông góc với $AI_b$ cắt nhau tại $P$. Chứng minh rằng $P$ thuộc một đường cố định khi $C$ thay đổi.

Bài 4. Cho $S$ là tập hợp khác rỗng và $A_1,A_2,\ldots,A_m\ (m\ge 2)$ là $m$ tập con của $S$. Gọi $\mathcal T$ là tập hợp gồm tất cả các tập hợp $A_i\Delta A_j\ (1\le i,j \le m$). Chứng minh rằng $|\mathcal T| \ge m$.

(Ký hiệu $A\Delta B=(A\backslash B)\cup (B\backslash A)$ là hiệu đối xứng của hai tập hợp $A,B$).

Giải

Bài 1.

Theo bất đẳng thức Cauchy-Schwarz, ta có

$$ \sum \dfrac{x^4}{x^3+y^2+z^2} \ge \dfrac{ \left( x^2+y^2+z^2 \right)^2}{ x^3+y^3+z^3+2 \left( x^2+y^2+z^2 \right)} $$

Cần chứng minh $\dfrac{ \left( x^2+y^2+z^2 \right)^2}{ x^3+y^3+z^3+2 \left( x^2+y^2+z^2 \right)} \ge \dfrac{1}{7} $ hay

$$7 \left( x^2+y^2+z^2 \right)^2 \ge x^3+y^3+z^3+2 \left( x^2+y^2+z^2 \right).$$ Ta có ${{(xy+yz+zx)}^{2}}\ge 3xyz(x+y+z)=3xyz$ và

$${{x}^{3}}+{{y}^{3}}+{{z}^{2}}-3xyz=(x+y+z)({{x}^{2}}+{{y}^{2}}+{{z}^{2}}-xy-yz-zx)$$ nên ${{x}^{3}}+{{y}^{3}}+{{z}^{3}}=3xyz+1-3(xy+yz+zx)\le {{(xy+yz+zx)}^{2}}+1-3(xy+yz+zx).$

Đặt $q=xy+yz+zx$ thì vì ${{(x+y+z)}^{2}}\ge 3(xy+yz+zx)$ nên $q\le \frac{1}{3}.$ Ta đưa về

$$7{{(1-2q)}^{2}}\ge {{q}^{2}}+1-3q+2(1-2q)$$ hay

$$(1-3q)(4-9q)\ge 0.$$

Do $q\le \frac{1}{3}$ nên $q\le \frac{4}{9}$ và bất đẳng thức trên là đúng. Vậy ta có đpcm.

Bài 2.

Nhận xét rằng vai trò của số $2017$ trong bài toán là không cần thiết cho nên ta sẽ giải bài toán khi thay $2017$ bởi số nguyên dương $p$ bất kỳ. Từ điều kiện đầu tiên, ta có được $f(p^k)=p^{3k}$ với $k$ là số nguyên dương bất kỳ.

Trong điều kiện thứ hai, thay $n$ bởi $m$, ta có $f(m)$ là bội của $m$ với mỗi $m$ nguyên dương nên ta đặt $f(m)=m.g(m)$ ($g:\mathbb{N^{*}}\rightarrow \mathbb{N^{*}}$). Khi đó ta có các điều kiện sau:

i/ $g(mn)=g(m).g(n) \forall m,n \in\mathbb{N^{*}}$

ii/ $mg(m)+ng(n)$ là bội của $m+n$.

iii/ $g(p^{n})=p^{2n} \forall n\in \mathbb{N^{*}}$.

Đặt $h(m)=g(m)-m^2$ ($h:\mathbb{N^{*}}\rightarrow \mathbb{Z}$) và thay $n$ bởi $p^n$ tại ii), ta có $m.h(m)$ là bội của $m+p^n$. Chọn $n$ đủ lớn thì $h(m)=0$ với mỗi $m$ hay $f(m)=m^3$ với mỗi $m$ nguyên dương. Thử lại thoả mãn.

Vậy $f(m)=m^3$ là nghiệm hàm duy nhất.

Bài 3.

(a) Trước hết, ta có một kết quả quen thuộc sau.

Bổ đề: Gọi $A_1$, $B_1$ lần lượt là điểm chính giữa các cung $BC$, $AC$ không chứa $A$, $B$ của $(O)$. Khi đó $A$, $I$, $A_1$, $I_a$ thẳng hàng và $A_1$ là trung điểm của $II_a$. Tương tự đối với $B$, $I$, $B_1$, $I_b$.

Trở lại bài toán, theo bổ đề, phép vị tự tâm $I$, tỉ số $2$ biến $\Delta OA_1B_1$ thành $\Delta MI_aI_b$, do đó tam giác này cân tại $M$.

Mở ảnh

(b) Ta thực hiện chuyển đổi mô hình. Gọi $I_a$ là tâm bàng tiếp góc $A$ của tam giác $ABC$ thì $(O)$ chính là đường tròn Euler của tam giác $I_aI_bI_c$. Xét bổ đề sau:

Bổ đề: Cho tam giác $ABC$ có đường thẳng $d$ đi qua tâm ngoại tiếp $O$. Gọi $D,E,F$ lần lượt là hình chiếu của $A,B,C$ lên $d$. Chứng minh rằng đường thẳng qua $D,E,F$ vuông góc với $BC,CA,AB$ đồng quy trên đường tròn $Euler$ của tam giác $ABC$.

Gọi $l$ là đường thẳng đi qua trực tâm $H$ của tam giác $ABC$ và vuông góc với $d$. Gọi $S$ là điểm anti-Steiner của $l$. $J$ là điểm đối xứng của $S$ qua $BC$ và $X$ là giao điểm của $SJ$ và $(O)$. $K$ là điểm đối xứng với $H$ qua $BC$. \medskip

Ta có: $$\angle AXS =\angle AKS=\angle KHJ$$ suy ra $HJ \parallel AX$. Do đó, $D$ nằm trên $AX$ hay $D$ là trung điểm $AX$. Suy ra đường thẳng qua $D$ vuông góc với $BC$ đi qua trung điểm $I$ của $SH$ và nằm trên đường tròn $Euler$ của tam giác $ABC$. \medskip

Trở lại bài toán, ta áp dụng bổ đề trên cho đường thẳng $OI$ đi qua tâm đường tròn $(I_aI_bI_c)$ thì dễ dàng có $P \in (O).$

Bài 4.

Ta sẽ chứng minh quy nạp theo $m$ cho điều này. Trước hết ta phát biểu bổ đề: $A\Delta B=A\Delta C$ thì $B=C$.

Giả sử $B\neq C$, khi đó không giảm tổng quát giả sử giả sử $a$ là phần tử thỏa $a\in B$ và $a\notin C$. Ta có hai trường hợp như sau:

  •  Nếu $a\in A$ khi đó $a\notin (A\setminus B),a\notin (B\setminus A)\Rightarrow a\notin A\Delta B$. Nhưng lại có $a\in (A\setminus C)$ nên suy ra $a\in A\Delta C$ nên $A\Delta B\neq A\Delta C$, vô lý.
  •  Nếu $a\notin A$ thì chứng minh tương tự suy ra $a\notin A\Delta C$ và $a\in A\Delta B$ nên suy ra $A\Delta B\neq A\Delta C$. Như vậy ta suy ra $B=C$.

Bây giờ ta sẽ quy nạp theo $m$. Với $m=1$ thì ta có một tập thuộc $T$ là tập rỗng. Với $m=2$ và hai tập $A,B$ thì ta có hai tập thuộc $T$ là tập rỗng và $A\Delta B$ thỏa. Như vậy giả thiết đúng với $m=1,2$.

Giả sử giả thiết đúng với $m=k$ thì ta chứng minh nó đúng với $m=k+1$. Xét $m+1$ tập $A_1,A_2,\ldots,A_{m+1}$. Nếu với $m$ tập $A_1,A_2,\ldots,A_m$ mà số lượng tập tạo thành không nhỏ hơn $m+1$ thì khi đó ta thêm vào một tập $A_{m+1}$ thì giả thiết vẫn đúng. Do đó ta chỉ xét cho trường hợp $|T|=m$.

Khi đó, nếu ta thêm vào một tập $A_{m+1}$ thì ta sẽ thêm vào tập $T$ các tập hợp $A_{m+1}\Delta A_1,\ldots,A_{m+1}\Delta A_{m+1}$. Nếu các tập này trùng với $m$ tập đã có trong $T$ thì do $|T|=m$ nên theo nguyên lý Dirichlet tồn tại $i,j,1\leq i<j\leq m+1$ để $A_{m+1}\Delta A_i=A_{m+1}\Delta A_j$ và theo bổ đề ta có $A_i=A_j$, vô lý. Vậy trong $m+1$ tập đó chắc chắn có một tập khác với các tập trong $T$ và số phần tử của $T$ tăng lên ít nhất một đơn vị, tức là $|T|\geq m+1$.

Vậy giả thiết quy nạp là đúng và ta có đpcm.

Giải bài toán bằng đại lượng cực biên – Phần 1

(Bài viết dành cho học sinh lớp 8,9 và đầu lớp 10)

Có một câu chuyện thú vị thường thấy là trong lớp học những người nào ngồi bàn đầu hay bàn cuối thì thường hay bị gọi lên bảng trả bài hơn là những người khác, vì sao như vậy? Thực sự vì hai vị trí đó là vị trí đầu và cuối, tức là vị trí biên, vị trí “đặc biệt” hơn các vị trí khác, nên dễ được chú ý hơn.

Hoặc có một bài toán đơn giản sau: Tam giác $ABC$, $M$ thuộc cạnh $BC$, với vị trí nào của $M$ thì $AM$ đạt giá trị lớn nhất? (nhỏ nhất?). Dễ nhận ra rằng $AM \leq AB$ hoặc $AM \leq AC$, do đó $AM$ lớn nhất chỉ khi $M$ là một trong hai vị trí $B$ hoặc $C$, đó chính là vị trí biên của đoạn thẳng.

Do đó các vị trí biên của một tập hợp $X$ nào đó luôn có những đặc điểm mà vị trí khác không có được, kiểu nếu lệch ra một tí thì “bay màu” khỏi $X$.

Nguyên lý cực biên cũng như nguyên lý quy nạp, đó là một trong các nguyên lý quan trọng để chứng minh các định lý hay các bài toán. Xuất phát tự quan hệ thứ tự trong tập các số thực, và tiên đề xây dựng số tự nhiên, ta có các tính chất sau

  • Mọi tập con khác rỗng hữu hạn của tập số thực luôn có phần tử lớn nhất và nhỏ nhất.
  • Mọi tập con khác rỗng của tập các số tự nhiên đều có phần tử nhỏ nhất
  • Mọi tập con khác rỗng bị chặn trên của tập số nguyên có phần tử lớn nhất, bị chặn dưới thì có phần tử nhỏ nhất.

Nguyên lý cực biên xuất hiện nhiều trong các chứng minh, trong bài viết nhỏ này tôi chỉ giới thiệu một số bài toán cơ bản thường gặp để giúp các em học sinh nắm được kĩ thuật chứng minh này, từ đó vận dụng để làm các bài toán khó hơn.

Việc sử dụng nguyên lí cực hạn có cái quan trọng nhất là mình sử dụng đặc điểm đặc biệt của đại lượng cực biên, xem như một giả thiết mới để khai thác, kết hợp với các kĩ thuật sắp xếp, phản chứng để giải quyết bài toán.

Ta xét vài ví dụ sau

Bài 1. Cho số thực $x$ chứng minh rằng tồn tại duy nhất số nguyên $n$ sao cho $n\leq x < n+1$. ($n$ được gọi là phần nguyên của $x$, kí hiệu là $[x]$.

Lời giải. 

Nhận xét: rõ ràng $n$ là số nguyên mà nhỏ hơn và “gần” $x$ nhất, tức là nếu $n$ tăng thêm một đơn vị thì nó sẽ vượt qua $x$. Từ ý đó ta có thể giải như sau:

Đặt $A = \{n \in \mathbb{Z}, n \leq x \}$, ta thấy $A$ là tập con khác rỗng của $\mathbb{Z}$, bị chặn trên bởi $x$ nên tồn tại phần tử lớn nhất, đặt là $n_\circ$. Ta chứng minh $n_\circ \leq x < n_\circ+1$.

Rõ ràng $n_\circ \in A$ nên $n_\circ \leq x$.

Giả sử $n_\circ + 1 \leq x$ thì $n_\circ \in A$ và $n_\circ + 1  > n_\circ $ vô lí vì $n_\circ$ là phần tử lớn nhất của $A$. Do đó $n_\circ +1 > x$

Từ đó ta có $n_\circ \leq x < n_\circ + 1$.

Bước kế tiếp là chứng minh duy nhất,giả sử tồn tại $n’$ nguyên thỏa $n’\leq x < n’+1$. \

Nếu $n’ > n_\circ$ thì $n’ \geq n_\circ+1 > x$, vô lí, tương tự với $n_\circ > n’$.

Do đó $n’ = n_\circ$.

Bài 2. Cho hai số nguyên dương $a, b$. Chứng minh rằng tồn tại duy nhất cặp số $q, r$ sao cho $0 \leq r \leq b-1$ và $$a = bq + r$$

Lời giải. Do $0 \leq r \leq b-1$ nên mình thấy rằng, $q$ trong đẳng thức trên là số lớn nhất để hiệu $a-bq$ không không âm.

Đặt $A = \{a-bq \leq 0, q\in \mathbb{N} \}$.

Rõ ràng $A$ khác rỗng vì $a-b \cdot 0 > 0$, và là tập con của tập các số tự nhiên. Khi đó $A$ có phần tử nhỏ nhất, đặt là $r$, ta có $q$ để $r = a-bq$. Ta chứng minh $0 \leq r \leq b-1$.

Rõ ràng $r \in A$ nên $r \geq 0$.

Ở ý còn lại, ta giả sử $r \geq b$, khi đó $r-b = a-bq-b = a-b(q+1) \geq 0$ và $r-b < r$, do đó $r-b$ thuộc $A$ và nhỏ hơn $r$,  mâu thuẫn với $r$ là số nhỏ nhất thuộc $A$.

Giả sử tồn tại cặp $q’, r’$ thỏa đề bài. Khi đó $a = bq+r = bq’+r’$

suy ra $r-r’ = b(q’-q)$ chia hết cho $b$ mà $|r-r’| \leq b-1$, do đó $r-r’=0$, và $q-q’=0$. Ta có điều cần chứng minh.

Ví dụ 3. Cho $a, b$ là hai số nguyên dương, gọi $d$ là ước chung lớn nhất của $a$ và $b$. Chứng minh rằng tồn tại các số nguyên $x, y$ thỏa $$d = x\cdot a + y \cdot b$$

Lời giải. Ý tưởng tương tự như bài trên, xét tập các tổ hợp tuyến tính dương của $a, b$ có dạng $xa + yb$,

Đặt T = ${xa + yb| x,y \in Z, xa +yb >0}$. Rõ ràng $T$ khác rỗng và là tập con của tập các số tự nhiên nên có phần tử nhỏ nhất, đặt là $e$.
Khi đó T có phần tử nhỏ nhất, ta đặt $e = xa + yb$.
Giả sử $a = ek +r$, với $ 0 \leq r < e$ , suy ra $r = a – ek = a – (xa +yb).k = a(1 – xk) + b. yk$.

  • Nếu $r >0$ thì $r \leq e$ mâu thuẫn vì $e$ là phần tử nhỏ nhất của $T$.
  • Vậy $r =0$ suy ra $e|a$. Chứng minh tương tự ta có $e|b$ do đó $e|d$.
  • Mặt khác $d|a, d|b$ suy ra $d|(xa + yb)$ hay $d|e$. Từ đó ta có $d = e$.

Ví dụ 4. Chứng minh rằng $\sqrt{2}$ là số vô tỉ.

Lời giải. Việc chứng minh $\sqrt{2}$ là số vô tỉ có nhiều cách, nhìn chung đều sử dụng phản chứng, và tính chất số học, lần này ta trình bày với phản chứng kết hợp với đại lượng cực biên.

Giả sử $\sqrt{2}$ không là số vô tỉ, tức là $\sqrt{2} = \dfrac{a}{b}$ trong đó $a, b$ là các số nguyên dương, suy ra $b\sqrt{2} = a$ là số nguyên dương.

Đặt $A = \{n| n, n\sqrt{2} \in \mathbb{N}\}$. Rõ ràng, $A$ khác rỗng là con của tập các số nguyên dương, nên có phần tử nhỏ nhất, đặt là $k$.

Ta có $k, k\sqrt{2}$ nguyên dương, suy ra $k(\sqrt{2}-1)$ nguyên dương.

Và $k(\sqrt{2}-1)\sqrt{2} = 2k – k\sqrt{2}$ cũng nguyên dương.

Do đó $k(\sqrt{2}-1)$ thuộc $A$ và $0 < k(\sqrt{2}-1) < k$ vô lí vì $k$ là nhỏ nhất.

Ví dụ 5. Chứng minh rằng không tồn tại các số nguyên dương $x, y, z, t$ sao cho $$x^2+y^2=3(z^2+t^2)$$

Lời giải. Giả sử tồn tại bộ 3 số nguyên dương thỏa đề bài, ta chọn bộ thỏa $x^2+y^2$ nhỏ nhất. Khi đó $x^2+y^2$ chia hết cho 3, suy ra $x, y$ đều chia hết cho $3$, khi đó $x= 3x’, y=3y’$, suy ra $z^2+t^2 = 3(x’^2+y’^2)$, thì bộ $(z,t,x’,y’)$ cũng thỏa đề bài, nhưng $z^2 +t^2 < x^2+y^2$. Mâu thuẫn.

Do đó phương trình không có nghiệm trong tập các số nguyên dương.

(Hết phần 1)

Tài liệu tham khảo. 

[1] Giải toán bằng phương pháp Đại lượng cực biên – Nguyễn Hữu Điển

[2] Problems Solving Strategies –

Đề thi và đáp án chọn đội tuyển toán trường PTNK năm 2021

Ngày thi thứ nhất. 

Bài 1. Tìm hàm số $f: \mathbb{R} \rightarrow \mathbb{R}$ thỏa $f(x f(y)+f(x))=f(x)+x y+x+1, \forall x, y \in \mathbb{R} .$

Bài 2. Cho dãy số $\left(u_{n}\right)$ thỏa $u_{1}=2, u_{2}=1$ và $u_{n+1}=\sqrt{\dfrac{u_{n} u_{n-1}}{n}}$ với mọi $n \geq 2$.
Xét dãy số $\left(v_{n}\right)$ xác định bởi $v_{n}:=u_{1}+u_{2}+\ldots+u_{n}, \forall n \geq 1$. Chứng minh dãy $\left(v_{n}\right)$ hội tụ.

Bài 3. Cho $p$ là số nguyên tố, $n$ là số nguyên dương thỏa $2<p<n$. Gọi $\mathrm{A}$ là tập hợp các đa thức $P(x)=x^{n}+a_{n-1} x^{n-1}+\ldots+a_{1} x+a_{0}$ có tất cả các hệ số thuộc tập ${1 ; 2 ; \ldots ; n !}$ và $P(m)$ chia hết cho $p$ với mọi số nguyên dương $m$.

a) Chứng minh tổng $a_{1}+a_{p}+a_{2 p-1}+\ldots+a_{1+k(p-1)}$ chia hết cho $p$ với mọi $k=\left[\dfrac{n-1}{p-1}\right]$ (xem $a_{n}=1$ ), kí hiệu $[x]$ là phần nguyên của $x$.
b) Tính số phần tử của $\mathrm{A}$ theo $\mathrm{n}$ và $\mathrm{p}$.

Bài 4. Cho tam giác $\mathrm{ABC}$ có (I) là đường tròn nội tiếp. Một đường thẳng qua $\mathrm{A}$ cắt $(\mathrm{I})$ tại $\mathrm{M}, \mathrm{N}$. Gọi $\mathrm{T}$ là giao điểm của các tiếp tuyến với (I) tại $\mathrm{M}, \mathrm{N}$.

b) Chứng minh rằng nếu $\mathrm{AT} \parallel \mathrm{BC}$ thì $\mathrm{MN}$ đi qua trung điểm $\mathrm{K}$ của $\mathrm{BC}$.
c) Gọi $\mathrm{D}$ là tiếp điểm của (I) với $\mathrm{AB}$ và $\mathrm{E}$ là giao điểm của $\mathrm{DM}$ với $\mathrm{AC}$. Trên $\mathrm{EN}$ lấy điểm $\mathrm{F}$ thoả $\mathrm{TF}$ vuông góc $\mathrm{AI}$. Chứng minh rằng khi đường thẳng $\mathrm{AMN}$ thay đổi, giao điểm $\mathrm{P}$ của $\mathrm{MF}$ và $\mathrm{DN}$ thuộc một đường thẳng cố định.

Ngày thi thứ hai

Bài 5. Cho $n$ số thực $x_{1}, x_{2}, \ldots, x_{n}$ thỏa hiệu giữa số lớn nhất và số nhỏ nhất của chúng là 1 . Ta xây dựng
$$
y_{1}=x_{1}, y_{2}=\frac{x_{1}+x_{2}}{2}, \ldots, y_{n}=\frac{x_{1}+x_{2}+\ldots+x_{n}}{n}
$$
Gọi $M, m$ lần lượt là số lớn nhất và nhỏ nhất trong các số $y_1, y_2,\cdots,y_n$. \
Tìm giá trị lớn nhất của $M-n$.

Bài 6.  Cho tập $\mathrm{X}={1 ; 2 ; \ldots ; 20}$. Tập con $\mathrm{A}$ của $\mathrm{X}$ được gọi là tập “tránh 2 ” nếu với mọi $\mathrm{x}, \mathrm{y}$ thuộc $\mathrm{A}$ thì $|x-y|$ khác 2 . Tìm số các tập con “tránh 2 ” của $\mathrm{X}$ có 5 phần tử.

Bài 7. Cho tam giác $\mathrm{ABC}$ và điểm $\mathrm{D}$ trên cạnh $\mathrm{BC}$. Các đường tròn ( $\mathrm{ABD}$ ), ( $\mathrm{ACD}$ ) lần lượt cắt $\mathrm{AC}, \mathrm{AB}$ tại $\mathrm{E}, \mathrm{F}$. Gọi $\mathrm{I}$ là tâm đường tròn $(\mathrm{AEF})$.
a) Chứng minh ID vuông góc BC.
b) Gọi $\mathrm{H}$ là giao điểm của $\mathrm{ID}$ với $\mathrm{EF}$ và $\mathrm{K}$ là điểm thoả mãn $H B K=H C K=90^{\circ}$. Các đường tròn (IBK), (ICK) lần lượt cắt IC, IB tại M, N. Chứng minh tâm J của đường tròn (IMN) thuộc trung trực BC.

Bài 8.  Cho $p$ là số nguyên tố. Với mọi số nguyên a, đặt
$$
q:=1+a+a^{2}+\ldots+a^{p-1} .
$$
Chứng minh $(1-a)\left(1-a^{2}\right) \ldots\left(1-a^{p-1}\right)-p$ chia hết cho $q$.

 

Đáp án sẽ được đăng trong Tập san Star education số 7/2022

Một số định lý, mô hình hình học quan trọng hình học 9

Bài 1. (Đường thẳng Euler, Đường tròn Euler) Cho tam giác $ABC$, các đường cao $AD, BE, CF$ cắt nhau tại $H$, trung điểm các cạnh là $M, N, P$, các đường thẳng $AM, BN, CP$ cắt nhau tại $G$. Gọi $O$ là tâm đường tròn ngoại tiếp tam giác $ABC$.

a) Chứng minh $AH = 2OM$.

b) Chứng minh $H, G, O$ thẳng hàng và $HG = 2OG$. (Đường thẳng qua $O, H, G$ là đường thẳng Euler)

c) Gọi $X, Y, Z$ là trung điểm của $HA, HB, HC$. Chứng minh 9 điểm $D, E, F, M, N, P, X, Y, Z$ cùng thuộc một đường tròn và tâm là trung điểm $OH$. (Đường tròn Euler – Đường tròn 9 điểm).

d) Lấy một điểm $T$ thuộc (O), chứng minh trung điểm của $HT$ thuộc đường tròn Euler.

Hướng dẫn

a) Vẽ đường kính $AK$, ta có $BHCK$ là hình bình hành, trung điểm $M$ của $BC$ cũng là trung điểm $HK$, tam giác $AHK$ thì $OM$ là đường trung bình nên $AH = 2OM$.

b) Tam giác $AHK$ có $AM$ là trung tuyến và $GA =2GM$ nên $G$ cũng là trọng tâm, do đó $H, G, O$ thẳng hàng và $HG = 2GO$.

c) Ta có $\angle XEH = \angle XHE, \angle MEH = \angle MBE$, suy ra $\angle MEX = \angle XEH + \angle MEH = \angle XHE + \angle MBE = 90^\circ$, suy ra $E$ thuộc đường tròn đường kính $XM$ tâm $J$.

$XN||CH, MN||AB$, suy ra $MN \bot NX$, suy ra $N$ thuộc $(J)$.

$MZ||BH, XZ ||AC$ suy ra $\angle MZX = 90^\circ$, suy ra $Z \in (J)$.

Từ đó chứng minh được các điểm cùng thuộc đường tròn đường kính $MX$.

$HXMO$ là hình bình hành nên $J$ là trung điểm $OH$.

d) Tam giác $MNP$ và $ABC$ đồng dạng, tỉ số 1/2 nên đường tròn Euler có bán kính bằng 1/2 bán kính đường tròn ngoại tiếp tam giác $ABC$.

Xét tam giác $HOT$ có $JL$ là đường trung bình nên $JL= \dfrac{1}{2}OT$, suy ra $L$ thuộc $(J)$.

Bài 2. (Đường thẳng Simson – Đường thẳng Steiner) Cho tam giác $ABC$ nội tiếp đường tròn $w$, $P$ là một điểm thuộc $(w)$. Gọi $D, E, F$ là hình chiếu của $P$ trên các đường thẳng $BC, AC, AB$.

a) Chứng minh rằng $D, E, F$ cùng thuộc một đường thẳng. (Đường thẳng Simson của tam giác $ABC$ ứng với $P$.

b) Gọi $D’, E’,F’$ đối xứng của $P$ qua $BC, AC, AB$. Chứng minh rằng $D’, E’, F’$ cùng thuộc một đường thẳng và đường thẳng này qua trực tâm của tam giác $ABC$.

Hướng dẫn

Bài 3. (Bài toán về điểm humpty) Cho tam giác $ABC$, các đường cao $AD, BE, CF$ cắt nhau tại $H$, $M$ là trung điểm $BC$, $P$ là hình chiếu của $H$ trên $AM. Khi đó

a) $P$ là giao điểm của đường tròn đường kính $AH$ và đường tròn ngoại tiếp tam giác $BHC$. ($P$ được gọi là điểm $A-humpty$)

b) $MP \cdot MA = MB^2 = \dfrac{1}{4}BC^2$ và $BC$ là tiếp tuyến chung của $(ABP)$ và $(ACP)$

c) Vẽ $AQ$ vuông góc $MH$, thì $Q$ thuộc $(ABC)$.

d) $AQ, HP, BC$ đồng quy.

Hướng dẫn

a) Ta có các $AP \cdot AM = AH \cdot AD = AF \cdot AB$, suy ra $BFPM$ nội tiếp. Khi đó $\angle MPB = \angle MFB = \angle ABM$.

Chứng minh tương tự thì $\angle MPC = \angle ACB$

Suy ra $\angle BPC = \angle MPB + \angle MPC = \angle B + \angle C = 180^\circ – \angle A = \angle BHC$.

Suy ra $BHPC$ nội tiếp.

b) Từ câu a, ta có $\angle MPB = \angle ABM$, suy ra tam giác $MPB$ và $MBA$ đồng dạng, khi đó $MA \cdot MP = MB^2 = \dfrac{1}{4} BC^2$.

c) Ta xét tam giác $BHC$ với $A$ là trực tâm thì vai trò điểm $Q$ giống vai trò điểm $P$, nên $Q$ thuộc đường tròn ngoại tiếp tam giác $ABC$.

d) Xét tam giác $AHM$ thì $AQ, HP, DM$ là 3 đường cao nên đồng quy.

Bài 4. (Tứ giác điều hòa – Điểm Dumpty).  Cho tam giác $ABC$ nội tiếp đường tròn $(O)$, tiếp tuyến tại $B, C$ cắt nhau tại $P$, $AP$ cắt $(O)$ tại $D$ khác $A$ và cắt $BC$ tại $I$. $OP$ cắt $BC$ tại $M$.

a) Chứng minh $OMDA$ nội tiếp và $\dfrac{IA}{ID} = \dfrac{PA}{PD}$

b) Chứng minh $\angle MDC = \angle ADB$ và $AD\cdot BC = 2 AC \cdot DB = 2 BD \cdot AC$.

c) Tiếp tuyến tại $A,D$ cắt nhau tại $Q$. Chứng minh $Q$ thuộc $BC$.

d) Gọi $X$ là giao điểm của $OQ$ và $AD$, chứng minh $\angle XBA = \angle XAC, \angle XAC = \angle XBA$. (Điểm $A-dumpty$ của tam giác $ABC$).

Hướng dẫn

a) $PM \cot PO = PB^2 = PA \cdot PD$.

$\angle PMD = \angle PAO = \angle ODA = \angle AMO$, suy ra $MP, MI$ là phân giác ngoài và phân giác trong của $\angle APD$.

b) $MO \cdot MP = MB^2 = MA \cdot MD$, suy ra $ABM$ và $BMD$ đồng dạng.

c) 5 điểm $A, P, M, D, Q$ cùng thuộc đường tròn, $QA = QD$ nên $MQ$ là phân giác $\angle AMD$.

d)  Chứng minh $BAX$ và $BCD$ đồng dạng, do $AX \cdot BC = AB \cdot CD$.

Bài 5. Cho tam giác $ABC$, có $O$ là tâm đường tròn ngoại tiếp tam giác. Một đường thẳng vuông góc với $OA$ cắt các cạnh $AB, AC$ tại $F, E$ và đường thẳng $BC$ tại $D$. 

a) Chứng minh $BFEC$ nội tiếp.

b) Đường tròn ngoại tiếp tam giác $AEF$ cắt $(O)$ tại điểm $P$ khác $A$. Chứng minh các tam giác $PEF$ và $PCB$ đồng dạng.

c) Chứng minh các tứ giác $BDPF, BCEP$ nội tiếp và $A, P, D$ thẳng hàng.

d) Gọi $O_a, O_b$ lần lượt là tâm đường tròn ngoại tiếp các tam giác $AEF, BDF$ và $BCEF$. Chứng minh $O_a, O_b, O_c, O$ cùng thuộc một đường tròn.

Hướng dẫn

a) Vẽ tiếp tuyến tại $A$ là $Ax$, $\angle ACB = \angle xAB = \angle AEF$.

b) $\angle AFP = \angle AEP, \angle PBA = \angle BCA$.

c) $\angle PEF = \angle PAC = \angle PBD$

$\angle DPF + \angle APF = \angle ABC + \angle CEF = 180^\circ$.

d) $O_bO_c$ là trung trực $BF, $O_aO_c$ là trung trực $EF$.

Suy ra $\angle O_aO_cO_b = \dfrac{1}{2} \angle $ACB$.

Tương tự cũng có $\angle O_aOO_b$

Bài 6. (Tứ giác điều hòa) xem tại đây https://geosiro.com/?p=1185

Đáp án đề thi chọn đội tuyển trường PTNK năm 2020

Ngày thi thứ nhất.

Bài 1. Với mỗi số nguyên dương $n$, tìm số thực $M_{n}$ lớn nhất sao cho với mọi số thực dương $x_{1}, x_{2}, \ldots, x_{n}$ thì ta đều có
$$
\sum_{k=1}^{n} \frac{1}{x_{k}^{2}}+\frac{1}{\left(\sum_{k=1}^{n} x_{k}\right)^{2}} \geq M_{n}\left(\sum_{k=1}^{n} \frac{1}{x_{k}}+\frac{1}{\sum_{k=1}^{n} x_{k}}\right)^{2}
$$

Bài 2. Cho 2021 số nguyên khác 0 . Biết rằng tổng của một số bất kỳ trong chúng với tích của tất cả 2020 số còn lại luôn âm.
(a) Chứng minh rằng với mọi cách chia 2021 số này thành hai nhóm và nhân các số cùng nhóm lại với nhau thì tổng của hai tích cũng luôn âm.
(b) Một bộ số thỏa mãn đề bài thì có thể có nhiều nhất mấy số âm?

Bài 3. Cho hai hàm số $f: \mathbb{R} \rightarrow \mathbb{R}$ và $g: \mathbb{R} \rightarrow \mathbb{R}$ thỏa mãn $g(2020)>0$ và với mọi $x, y \in \mathbb{R}$ thì $\left\{\begin{array}{l}f(x-g(y))=f(-x+2 g(y))+x g(y)-6 \\ g(y)=g(2 f(x)-y)\end{array}\right.$

(a) Chứng minh rằng $g$ là hàm hằng.

(b) Chứng minh rằng đồ thị $h(x)=f(x)-x$ nhận $x=1$ là trục đối xứng.

Bài 4. Cho tam giác $A B C$ nhọn, nội tiếp trong đường tròn $(O)$ có trực tâm $H$ và $A H, B H, C H$ cắt cạnh đối diện lần lượt tại $D, E, F$. Gọi $I, M, N$ lần lượt là trung điểm các cạnh $B C, H B, H C$ và $B H, C H$ cắt lại $(O)$ theo thứ tự tại các diểm $L, K$. Giả sử $K L$ cắt $M N$ ở $G$.
(a) Trên $E F$, lấy điểm $T$ sao cho $A T$ vuông góc với $H I$. Chứng minh rằng $G T$ vuông góc với $O H$.
(b) Gọi $P, Q$ lần lượt là giao điểm của $D E, D F$ và $M N$. Gọi $S$ là giao điểm của $B Q, C P$. Chứng minh rằng $H S$ di qua trung điểm của $E F$.

Ngày thi thứ hai.
Bài 5. Cho số nguyên dương $n>1$. Chứng minh rằng với mọi số thực $a \in\left(0 ; \frac{1}{n}\right)$ và mọi đa thức $P(x)$ có bậc $2 n-1$ thỏa mãn điều kiện $P(0)=P(1)=0$, luôn tồn tại các số thực $x_{1}, x_{2}$ thuộc $[0 ; 1]$ sao cho $P\left(x_{1}\right)=P\left(x_{2}\right)$ và $x_{2}-x_{1}=a$.

Bài 6. Giải phương trình sau trên $\mathbb{Z}^{+}:\left(x^{2}+3\right)^{3^{x+1}}\left[\left(x^{2}+3\right)^{3^{x+1}}+1\right]+x^{2}+y=x^{2} y$.

Bài 7 . Cho các số nguyên $n>k>t>0$ và $X={1,2, \ldots, n}$. Gọi $\mathcal{F}$ là họ các tập con có $k$ phần tử của tập hợp $X$ sao cho với mọi $F, F^{\prime} \in \mathcal{F}$ thì $\left|F \cap F^{\prime}\right| \geq t$. Giả sử không có tập con có $t$ phần tử nào chứa trong tất cả các tập $F \in \mathcal{F}$.
(a) Chứng minh rằng tồn tại một tập hợp $B \subset X$ sao cho $|B|<3 k$ và $|B \cap F| \geq t+1$ với mọi $F \in \mathcal{F}$.
(b) Chứng minh rằng $|\mathcal{F}|<C_{3 k}^{t+1} C_{n}^{k-t-1}$.

Bài 8. Cho tam giác $A B C$ nội tiếp trong $(O)$ với $B, C$ cố định và $A$ thay đổi trên cung lớn $B C$. Dựng hình bình hành $A B D C$ và $A D$ cắt lại $(B C D)$ ở $K$.
(a) Gọi $R_{1}, R_{2}$ lần lượt là bán kính đường tròn ngoại tiếp $(K A B),(K A C)$. Chứng minh rằng tích $R_{1} R_{2}$ không đổi.
(b) Ký hiệu $(T),\left(T^{\prime}\right)$ lần lượt là các đường tròn cùng đi qua $K$, tiếp xúc với $B D$ ở $B$ và tiếp xúc với $C D$ ở $C$. Giả sử $(T),\left(T^{\prime}\right)$ cắt nhau ở $L \neq K$. Chứng minh rằng $A L$ luôn đi qua một điểm cố định.

Hết